Math, asked by kumaripooja45643, 1 month ago

[(0.7)⁴-(0.5)⁴] / [(0.7)³-(0.5)³]​

Answers

Answered by MaheswariS
1

\textbf{Given:}

\mathsf{\dfrac{(0.7)^4-(0.5)^4}{(0.7)^3-(0.5)^3}}

\textbf{To simplify:}

\mathsf{\dfrac{(0.7)^4-(0.5)^4}{(0.7)^3-(0.5)^3}}

\textbf{Solution:}

\textsf{Consider,}

\mathsf{\dfrac{(0.7)^4-(0.5)^4}{(0.7)^3-(0.5)^3}}

\mathsf{Take\;a=0.7\;and\;b=0.5}

\mathsf{=\dfrac{a^4-b^4}{a^3-b^3}}

\mathsf{=\dfrac{(a^2)^2-(b^2)^2}{a^3-b^3}}

\mathsf{=\dfrac{(a^2-b^2)(a^2+b^2)}{a^3-b^3}}

\mathsf{=\dfrac{(a-b)(a+b)(a^2+b^2)}{(a-b)(a^2+ab+b^2)}}

\mathsf{=\dfrac{(a+b)(a^2+b^2)}{(a^2+ab+b^2)}}

\mathsf{=\dfrac{(a+b)((a+b)^2-2ab)}{(a+b)^2-ab}}

\mathsf{=\dfrac{(0.7+0.5)((0.7+0.5)^2-2(0.7)(0.5))}{(0.7+0.5)^2-(0.7)(0.5)}}

\mathsf{=\dfrac{(1.2)(1.2^2-2(0.35))}{1.2^2-(0.35)}}

\mathsf{=\dfrac{(1.2)(1.44-0.7)}{1.44-0.35}}

\mathsf{=\dfrac{(1.2)(0.74)}{1.09}}

\mathsf{=\dfrac{0.888}{1.09}}

\mathsf{=0.815}

\implies\boxed{\mathsf{\dfrac{(0.7)^4-(0.5)^4}{(0.7)^3-(0.5)^3}=0.815}}

\textbf{Find more:}

Evaluate the following:

(24/5)to the power -4(/12/15)to the power 6 *(6/4) to the power 5

https://brainly.in/question/15872264

Similar questions